LSAT and Law School Admissions Forum

Get expert LSAT preparation and law school admissions advice from PowerScore Test Preparation.

 Administrator
PowerScore Staff
  • PowerScore Staff
  • Posts: 8917
  • Joined: Feb 02, 2011
|
#40917
Complete Question Explanation
(The complete setup for this game can be found here: lsat/viewtopic.php?t=1007)

The correct answer choice is (D)

If Alicia takes S3 and G, the initial diagram appears as:
PT59_Game_#3_#15_diagram 1.png
Because G is selected, from the fifth rule W cannot be selected, and answer choice (E) can be eliminated.

Because P requires S9 from the fourth rule, and Alicia already takes S3 (which from the condition in the game scenario eliminates her ability to take S9), P cannot be selected. Therefore, answer choice (C) can be eliminated.

The remaining two incorrect answers are more difficult to eliminate. Consider for a moment the courses that have been eliminated:


..... ..... ..... ..... ..... W, P, and S9 each cannot be selected.


Thus, three courses have already been eliminated, which in an 8-into-4 scenario means that four of the remaining five courses must be selected. Let us examine those remaining five courses more closely. They are:

..... ..... ..... ..... ..... S3, G, J, R, and M


S3 and G are already selected. The remaining two selections must then come from the group of J, R, and M (remember, both J and R can be selected; the first rule just addresses what occurs when one is not selected). However, from the second rule, J and M cannot be selected, and thus, by Hurdling the Uncertainty, we can infer that R must be selected. Thus, answer choice (D) must be true and is correct.
 SherryZ
  • Posts: 124
  • Joined: Oct 06, 2013
|
#13062
Hi there, thank you guys for your generous help!

Dec 2009 LSAT, Sec 1 LG, Game 3, Q15:

I don't know why D is the correct answer?

Base on the question stem, since G is in, then W CANNOT be in. In addition, P CANNOT be in because if P is in, then Alicia will take S at 9am, which violates the question stem. Now, there are 2 remain spots. I don't know how to decide which class must to be taken :(

Please help me out and if my logic above is wrong, please correct me.

Thank you so much! Have a nice day!

---Sherry
User avatar
 KelseyWoods
PowerScore Staff
  • PowerScore Staff
  • Posts: 1079
  • Joined: Jun 26, 2013
|
#13070
Hi Sherry!

So you're right that W and P are definitely out. That leaves us with two available spots and 3 options to fill them: M, R, J.

By combining the first two rules, we learn that if she takes M, she can't take J, (second rule) and if she can't take J, she must take R (contrapositive of the first rule).

So: M :arrow: /J :arrow: R
M :arrow: R
contrapositive: /R :arrow: /M

Since we now know that if we have M, we have to have R, that means that R has to be in. With W and P out, there's only one more "out" slot. If R is out, M would have to be out as well, but we don't have enough out slots for that. So R must be in. For the final "in" slot, it could be M or J, either one would work.

That gives us answer choice (D): Alicia must take Russian.

Hope that helps!

Best,
Kelsey
 Basia W
  • Posts: 108
  • Joined: Jun 19, 2014
|
#15957
Good afternoon,

I went through this question and am at a loss of why Russian is the topic she must take. If she takes statistics at 3, and G, we know that she cannot have W. But because NOT W is at the end of the P :arrow: S9 chain we cannot infer any further.

Thank you for your help!

Best,

Basia
 Emily Haney-Caron
PowerScore Staff
  • PowerScore Staff
  • Posts: 577
  • Joined: Jan 12, 2012
|
#16000
We know she definitely takes: S3, G
We know she definitely cannot take: S9, H
We now need to use 2 of P, J, M, and R to fill her schedule. If she takes P, she has to take S9, and we know she can't - so now we know P is also out. That leaves J, M, and R. She cannot take both J and M together, so she HAS to take R.
 Jeanel
  • Posts: 2
  • Joined: Jul 05, 2016
|
#27525
Hi!

I just had a quick question about this. I was able to get this question right, but I was just wondering if someone could explain to me why B) was an incorrect answer? I am probably overthinking it but I just want to make sure. I was able to narrow down that R, J, and M were the pieces that had to fit into the group, but aren't R and M two pieces that equally MUST be chosen? Since if you choose M, you must choose R and if you choose R you must choose M for this specific situation? If J is chosen, the course schedule would not be filled.

Thank you so much!
 Clay Cooper
PowerScore Staff
  • PowerScore Staff
  • Posts: 241
  • Joined: Jul 03, 2015
|
#27546
Hi Jeanel,

Thanks for your question, and welcome to the forum! I hope you will find it useful.

It looks to me like you have misinterpreted a rule here. I'm not sure what you mean when you say that R, J, and M must be included; in fact, J and M can never both be included (due to the second rule: M :arrow: ~J). While it is true that, If you choose M, you must choose R as well, the reverse is not true; that is, if you choose R, it is not the case that you must choose M.

Choosing R is not the sufficient condition in any of our rules. We can deduce from the local rule that we must leave out W (because G is chosen) and the early Statistics (because the late statistics is chosen); furthermore, we know that since the early statistics is left out, Psychology must be too. That leaves only one open space in the out-group, which we know from the second rule must be filled by either J or M. There is no reason to suppose, however, that it could not be M in this space; in other words, we could choose G, the late Statistics, J, and R, without breaking any rules.

Does that help?

Get the most out of your LSAT Prep Plus subscription.

Analyze and track your performance with our Testing and Analytics Package.